Recent content by Jezzybean

  1. J

    Integrate e^(-x) dx between 0 and +infinity?

    [-e^(-x)] between 0 and +infinity = 1 - (e^(-infinity)) Right? My mind has gone blank. What do I do now?
Back
Top